Teorema di Helmholtz: differenze tra le versioni

Contenuto cancellato Contenuto aggiunto
Atarubot (discussione | contributi)
m fix parametri in template cita using AWB
Riga 22:
Scrivendo esplicitamente i potenziali si ha la ''decomposizione di Helmholtz'':
 
:<math>\mathbf{F} (\mathbf{r}) = - \frac{1}{4\pi} \, \nabla \left(\int_V{ \frac{\nabla' \cdot \mathbf{F}(\mathbf{r'})} {\left| \mathbf{r} - \mathbf{r'} \right|} \, \operatorname dV'} \right) +\frac{1}{4\pi} \, \nabla \times \left( \int_V{ \frac{\nabla' \times \mathbf{F}(\mathbf{r'})} {\left| \mathbf{r} - \mathbf{r'} \right|} \, \operatorname dV'} \right)</math>
 
dove l'operatore [[nabla]] agisce rispetto alle coordinate <math>\mathbf {r'}</math> all'interno degli integrali e rispetto alle coordinate <math>\mathbf r</math> all'esterno. Inoltre, l'integrazione avviene sulle coordinate <math>\mathbf {r'}</math>.
Riga 32:
allora <math>\mathbf{F}</math> è completamente determinato dalla sua divergenza e dal suo rotore:
 
:<math> \mathbf{F} (\mathbf{r}) = - \frac{1}{4\pi} \, \int_V{ \frac{\nabla \frac{(\nabla' \cdot \mathbf{F}(\mathbf{r'}))} {\left| \mathbf{r} - \mathbf{r'} \right|} \; \operatorname dV'} +\frac{1}{4\pi} \, \int_V{ \frac{\nabla \times \frac{(\nabla' \times \mathbf{F}(\mathbf{r'}))} {\left| \mathbf{r} - \mathbf{r'} \right|} \; \operatorname dV'} </math>
 
==Formulazione debole==